RH Diabetes Mellitus - NCLEX

¡Supera tus tareas y exámenes ahora con Quizwiz!

Which statement by a nurse to a patient newly diagnosed with type 2 diabetes is correct? a. Insulin is not used to control blood glucose in patients with type 2 diabetes. b. Complications of type 2 diabetes are less serious than those of type 1 diabetes. c. Changes in diet and exercise may control blood glucose levels in type 2 diabetes. d. Type 2 diabetes is usually diagnosed when the patient is admitted with a hyperglycemic coma.

ANS: C For some patients with type 2 diabetes, changes in lifestyle are sufficient to achieve blood glucose control. Insulin is frequently used for type 2 diabetes, complications are equally severe as for type 1 diabetes, and type 2 diabetes is usually diagnosed with routine laboratory testing or after a patient develops complications such as frequent yeast infections.

The health care provider suspects the Somogyi effect in a 50-year-old patient whose 6:00 AM blood glucose is 230 mg/dL. Which action will the nurse teach the patient to take? a. Avoid snacking at bedtime. b. Increase the rapid-acting insulin dose. c. Check the blood glucose during the night d. Administer a larger dose of long-acting insulin.

ANS: C If the Somogyi effect is causing the patient's increased morning glucose level, the patient will experience hypoglycemia between 2:00 and 4:00 AM. The dose of insulin will be reduced, rather than increased. A bedtime snack is used to prevent hypoglycemic episodes during the night.

When a patient with type 2 diabetes is admitted for a cholecystectomy, which nursing action can the nurse delegate to a licensed practical/vocational nurse (LPN/LVN)? a. Communicate the blood glucose level and insulin dose to the circulating nurse in surgery. b. Discuss the reason for the use of insulin therapy during the immediate postoperative period. c. Administer the prescribed lispro (Humalog) insulin before transporting the patient to surgery. d. Plan strategies to minimize the risk for hypoglycemia or hyperglycemia during the postoperative period.

ANS: C LPN/LVN education and scope of practice includes administration of insulin. Communication about patient status with other departments, planning, and patient teaching are skills that require RN education and scope of practice.

A patient who was admitted with diabetic ketoacidosis secondary to a urinary tract infection has been weaned off an insulin drip 30 minutes ago. The patient reports feeling lightheaded and sweaty. Which action should the nurse take first? a. Infuse dextrose 50% by slow IV push. b. Administer 1 mg glucagon subcutaneously. c. Obtain a glucose reading using a finger stick. d. Have the patient drink 4 ounces of orange juice.

ANS: C The patient's clinical manifestations are consistent with hypoglycemia and the initial action should be to check the patient's glucose with a finger stick or order a stat blood glucose. If the glucose is low, the patient should ingest a rapid-acting carbohydrate, such as orange juice. Glucagon or dextrose 50% might be given if the patient's symptoms become worse or if the patient is unconscious

After change-of-shift report, which patient will the nurse assess first? a. 19-year-old with type 1 diabetes who was admitted with possible dawn phenomenon b. 35-year-old with type 1 diabetes whose most recent blood glucose reading was 230 mg/dL c. 60-year-old with hyperosmolar hyperglycemic syndrome who has poor skin turgor and dry oral mucosa d. 68-year-old with type 2 diabetes who has severe peripheral neuropathy and complains of burning foot pain

ANS: C The patient's diagnosis of HHS and signs of dehydration indicate that the nurse should rapidly assess for signs of shock and determine whether increased fluid infusion is needed. The other patients also need assessment and intervention but do not have life-threatening complications.

A 26-year-old patient with diabetes rides a bicycle to and from work every day. Which site should the nurse teach the patient to administer the morning insulin? a. thigh. b. buttock. c. abdomen. d. upper arm.

ANS: C Patients should be taught not to administer insulin into a site that will be exercised because exercise will increase the rate of absorption. The thigh, buttock, and arm are all exercised by riding a bicycle

Which information is most important for the nurse to report to the health care provider before a patient with type 2 diabetes is prepared for a coronary angiogram? a. The patient's most recent HbA1C was 6.5%. b. The patient's admission blood glucose is 128 mg/dL. c. The patient took the prescribed metformin (Glucophage) today. d. The patient took the prescribed captopril (Capoten) this morning.

ANS: C To avoid lactic acidosis, metformin should be discontinued a day or 2 before the coronary arteriogram and should not be used for 48 hours after IV contrast media are administered. The other patient data will also be reported but do not indicate any need to reschedule the procedure.

After the nurse has finished teaching a patient who has a new prescription for exenatide (Byetta), which patient statement indicates that the teaching has been effective? a. "I may feel hungrier than usual when I take this medicine." b. "I will not need to worry about hypoglycemia with the Byetta." c. "I should take my daily aspirin at least an hour before the Byetta." d. "I will take the pill at the same time I eat breakfast in the morning."

ANS: C Since exenatide slows gastric emptying, oral medications should be taken at least an hour before the exenatide to avoid slowing absorption. Exenatide is injected and increases feelings of satiety. Hypoglycemia can occur with this medication.

The nurse is assessing a 22-year-old patient experiencing the onset of symptoms of type 1 diabetes. Which question is most appropriate for the nurse to ask? a. "Are you anorexic?" b. "Is your urine dark colored?" c. "Have you lost weight lately?" d. "Do you crave sugary drinks?"

ANS: C Weight loss occurs because the body is no longer able to absorb glucose and starts to break down protein and fat for energy. The patient is thirsty but does not necessarily crave sugar-containing fluids. Increased appetite is a classic symptom of type 1 diabetes. With the classic symptom of polyuria, urine will be very dilute.

A patient with type 2 diabetes is scheduled for a follow-up visit in the clinic several months from now. Which test will the nurse schedule to evaluate the effectiveness of treatment for the patient? a. Urine dipstick for glucose b. Oral glucose tolerance test c. Fasting blood glucose level d. Glycosylated hemoglobin level

ANS: D The glycosylated hemoglobin (A1C or HbA1C) test shows the overall control of glucose over 90 to 120 days. A fasting blood level indicates only the glucose level at one time. Urine glucose testing is not an accurate reflection of blood glucose level and does not reflect the glucose over a prolonged time. Oral glucose tolerance testing is done to diagnose diabetes, but is not used for monitoring glucose control once diabetes has been diagnosed.

Which action by a patient indicates that the home health nurse's teaching about glargine and regular insulin has been successful? a. The patient administers the glargine 30 minutes before each meal. b. The patient's family prefills the syringes with the mix of insulins weekly. c. The patient draws up the regular insulin and then the glargine in the same syringe. d. The patient disposes of the open vials of glargine and regular insulin after 4 weeks.

ANS: D Insulin can be stored at room temperature for 4 weeks. Glargine should not be mixed with other insulins or prefilled and stored. Short-acting regular insulin is administered before meals, while glargine is given once daily.

The nurse is interviewing a new patient with diabetes who receives rosiglitazone (Avandia) through a restricted access medication program. What is most important for the nurse to report immediately to the health care provider? a. The patient's blood pressure is 154/92. b. The patient has a history of emphysema. c. The patient's blood glucose is 86 mg/dL. d. The patient has chest pressure when walking.

ANS: D Rosiglitazone can cause myocardial ischemia. The nurse should immediately notify the health care provider and expect orders to discontinue the medication. There is no urgent need to discuss the other data with the health care provider.

Which finding indicates a need to contact the health care provider before the nurse administers metformin (Glucophage)? a. The patient's blood glucose level is 174 mg/dL. b. The patient has gained 2 lb (0.9 kg) since yesterday. c. The patient is scheduled for a chest x-ray in an hour. d. The patient's blood urea nitrogen (BUN) level is 52 mg/dL.

ANS: D The BUN indicates possible renal failure, and metformin should not be used in patients with renal failure. The other findings are not contraindications to the use of metformin.

The nurse has been teaching a patient with type 2 diabetes about managing blood glucose levels and taking glipizide (Glucotrol). Which patient statement indicates a need for additional teaching? a. "If I overeat at a meal, I will still take the usual dose of medication." b. "Other medications besides the Glucotrol may affect my blood sugar." c. "When I am ill, I may have to take insulin to control my blood sugar." d. "My diabetes won't cause complications because I don't need insulin."

ANS: D The patient should understand that type 2 diabetes places the patient at risk for many complications and that good glucose control is as important when taking oral agents as when using insulin. The other statements are accurate and indicate good understanding of the use of glipizide.

A 28-year-old male patient with type 1 diabetes reports how he manages his exercise and glucose control. Which behavior indicates that the nurse should implement additional teaching? a. The patient always carries hard candies when engaging in exercise. b. The patient goes for a vigorous walk when his glucose is 200 mg/dL. c. The patient has a peanut butter sandwich before going for a bicycle ride. d. The patient increases daily exercise when ketones are present in the urine.

ANS: D When the patient is ketotic, exercise may result in an increase in blood glucose level. Type 1 diabetic patients should be taught to avoid exercise when ketosis is present. The other statements are correct.

The nurse is evaluating a 45-year-old patient diagnosed with type 2 diabetes mellitus. Which symptom reported by the patient is considered one of the classic clinical manifestations of diabetes? a. Excessive thirst b. Gradual weight gain c. Overwhelming fatigue d. Recurrent blurred vision

a. Excessive thirst The classic symptoms of diabetes are polydipsia (excessive thirst), polyuria, (excessive urine output), and polyphagia (increased hunger). Weight gain, fatigue, and blurred vision may all occur with type 2 diabetes, but are not classic manifestations.

The health care provider orders oral glucose tolerance testing for a patient seen in the clinic. Which information from the patient's health history is most important for the nurse to communicate to the health care provider? a. The patient had a viral illness 2 months ago. b. The patient uses oral contraceptives. c. The patient runs several days a week. d. The patient has a family history of diabetes.

B Rationale: Oral contraceptive use may falsely elevate oral glucose tolerance test (OGTT) values. A viral 2 months previously illness may be associated with the onset of type 1 diabetes but will not falsely impact the OGTT. Exercise and a family history of diabetes both can affect blood glucose but will not lead to misleading information from the OGTT. Cognitive Level: Application Text Reference: p. 1267 Nursing Process: Assessment NCLEX: Physiological Integrity

A diabetic patient is started on intensive insulin therapy. The nurse will plan to teach the patient about mealtime coverage using _____ insulin. a. NPH b. lispro c. detemir d. glargine

B Rationale: Rapid or short acting insulin is used for mealtime coverage for patients receiving intensive insulin therapy. NPH, glargine, or detemir will be used as the basal insulin. Cognitive Level: Application Text Reference: p. 1260 Nursing Process: Planning NCLEX: Physiological Integrity

A patient receives a daily injection of 70/30 NPH/regular insulin premix at 7:00 AM. The nurse expects that a hypoglycemic reaction is most likely to occur between a. 8:00 and 10:00 AM. b. 4:00 and 6:00 PM. c. 7:00 and 9:00 PM. d. 10:00 PM and 12:00 AM.

B Rationale: The greatest insulin effect with this combination occurs mid afternoon. The patient is not at a high risk at the other listed times, although hypoglycemia may occur. Cognitive Level: Comprehension Text Reference: p. 1260 Nursing Process: Evaluation NCLEX: Physiological Integrity

Which of these laboratory values noted by the nurse when reviewing the chart of a diabetic patient indicates the need for further assessment of the patient? a. Fasting blood glucose of 130 mg/dl b. Noon blood glucose of 52 mg/dl c. Glycosylated hemoglobin of 6.9% d. Hemoglobin A1C of 5.8%

B Rationale: The nurse should assess the patient with a blood glucose level of 52 mg/dl for symptoms of hypoglycemia, and give the patient some carbohydrate-containing beverage such as orange juice. The other values are within an acceptable range for a diabetic patient. Cognitive Level: Application Text Reference: pp. 1281-1282 Nursing Process: Assessment NCLEX: Physiological Integrity

A patient with type 2 diabetes has sensory neuropathy of the feet and legs and peripheral vascular disease evidenced by decreased peripheral pulses and dependent rubor. The nurse teaches the patient that a. the feet should be soaked in warm water on a daily basis. b. flat-soled leather shoes are the best choice to protect the feet from injury. c. heating pads should always be set at a very low temperature. d. over-the-counter (OTC) callus remover may be used to remove callus and prevent pressure.

B Rationale: The patient is taught to avoid high heels and that leather shoes are preferred. The feet should be washed, but not soaked, in warm water daily. Heating pad use should be avoided. Commercial callus and corn removers should be avoided; the patient should see a specialist to treat these problems. Cognitive Level: Application Text Reference: p. 1287 Nursing Process: Implementation NCLEX: Health Promotion and Maintenance

A newly diagnosed type 1 diabetic patient likes to run 3 miles several mornings a week. Which teaching will the nurse implement about exercise for this patient? a. "You should not take the morning NPH insulin before you run." b. "Plan to eat breakfast about an hour before your run." c. "Afternoon running is less likely to cause hypoglycemia." d. "You may want to run a little farther if your glucose is very high."

B Rationale: Blood sugar increases after meals, so this will be the best time to exercise. NPH insulin will not peak until mid-afternoon and is safe to take before a morning run. Running can be done in either the morning or afternoon. If the glucose is very elevated, the patient should postpone the run. Cognitive Level: Application Text Reference: p. 1269 Nursing Process: Implementation NCLEX: Physiological Integrity

A patient with newly diagnosed type 2 diabetes mellitus asks the nurse what "type 2" means in relation to diabetes. The nurse explains to the patient that type 2 diabetes differs from type 1 diabetes primarily in that with type 2 diabetes a. the patient is totally dependent on an outside source of insulin. b. there is decreased insulin secretion and cellular resistance to insulin that is produced. c. the immune system destroys the pancreatic insulin-producing cells. d. the insulin precursor that is secreted by the pancreas is not activated by the liver.

B Rationale: In type 2 diabetes, the pancreas produces insulin, but the insulin is insufficient for the body's needs or the cells do not respond to the insulin appropriately. The other information describes the physiology of type 1 diabetes. Cognitive Level: Application Text Reference: p. 1255 Nursing Process: Implementation NCLEX: Physiological Integrity

Amitriptyline (Elavil) is prescribed for a diabetic patient with peripheral neuropathy who has burning foot pain occurring mostly at night. Which information should the nurse include when teaching the patient about the new medication? a. Amitriptyline will help prevent the transmission of pain impulses to the brain. b. Amitriptyline will improve sleep and make you less aware of nighttime pain. c. Amitriptyline will decrease the depression caused by the pain. d. Amitriptyline will correct some of the blood vessel changes that cause pain.

A Rationale: Tricyclic antidepressants decrease the transmission of pain impulses to the spinal cord and brain. Tricyclics also improve sleep quality and are used for depression, but that is not the major purpose for their use in diabetic neuropathy. The blood vessel changes that contribute to neuropathy are not affected by tricyclics. Cognitive Level: Application Text Reference: p. 1285 Nursing Process: Implementation NCLEX: Physiological Integrity

When assessing the patient experiencing the onset of symptoms of type 1 diabetes, which question should the nurse ask? a. "Have you lost any weight lately?" b. "Do you crave fluids containing sugar?" c. "How long have you felt anorexic?" d. "Is your urine unusually dark-colored?"

A Rationale: Weight loss occurs because the body is no longer able to absorb glucose and starts to break down protein and fat for energy. The patient is thirsty but does not necessarily crave sugar- containing fluids. Increased appetite is a classic symptom of type 1 diabetes. With the classic symptom of polyuria, urine will be very dilute. Cognitive Level: Application Text Reference: pp. 1255, 1258 Nursing Process: Assessment NCLEX: Physiological Integrity

A patient with type 2 diabetes is scheduled for an outpatient coronary arteriogram. Which information obtained by the nurse when admitting the patient indicates a need for a change in the patient's regimen? a. The patient's most recent hemoglobin A1C was 6%. b. The patient takes metformin (Glucophage) every morning. c. The patient uses captopril (Capoten) for hypertension. d. The patient's admission blood glucose is 128 mg/dl.

B Rationale: To avoid lactic acidosis, metformin should not be used for 48 hours after IV contrast media are administered. The other patient data indicate that the patient is managing the diabetes appropriately. Cognitive Level: Application Text Reference: p. 1266 Nursing Process: Assessment NCLEX: Physiological Integrity

** What is a typical finding of hyperosmolar hyperglycemic syndrome (HHS)? A. Occurs in type 1 diabetes as the presenting symptom B. Slow onset resulting in a blood glucose level greater than 600 mg/dL C. Ketone bodies higher than 4+ in urine D. Signs and symptoms of diabetes insipidus

B. Slow onset resulting in a blood glucose level greater than 600 mg/dL HHS has a slower onset than diabetic ketoacidosis. HHS is often related to impaired thirst sensation, inadequate fluid intake, or functional inability to replace fluids. Because of the slower onset, the blood glucose levels can be quite high (more than 600 mg/dL) before diagnosis. HHS is seen in type 2 diabetics, and there is enough circulating insulin to prevent ketoacidosis. Diabetes insipidus is related to inadequate antidiuretic hormone secretion or kidney response with dilute, frequent urination. It is not related to HHS.

Which elevated laboratory finding is the best indication of potential diabetic nephropathy? A. Blood urea nitrogen (BUN) level B. Urine albumin-to-creatinine ratio C. Urine specific gravity D. Chloride (Cl-) level

B. Urine albumin-to-creatinine ratio Screening for nephropathy depends on the urinary albumin-to-creatinine ratio and a serum creatinine level. BUN alone, without correction to creatinine, can indicate many other issues, including dehydration and liver function. Unless there is renal failure, urine specific gravity is more indicative of dehydration.

The patient has diabetes mellitus and macroalbuminuria. The patient asks you why the physician is prescribing the angiotensin-converting enzyme (ACE) inhibitor lisinopril (Zestril) for him even though his blood pressure is well-controlled. What is your response? A. It helps prevent hypertension as diabetics are prone to it. B. ACE inhibitors delay the progression of nephropathy in patients with diabetes. C. ACE inhibitors prevent macrovascular complications. D. ACE inhibitors help prevent atherosclerosis.

B. ACE inhibitors delay the progression of nephropathy in patients with diabetes. ACE inhibitors and angiotensin II receptor antagonists (ARBs) are used to treat hypertension and delay the progression of nephropathy in patients with diabetes. ACE inhibitors are not used prophylactically. ACE inhibitors do not affect macrovascular complications. Nephropathy is a microvascular complication.

The patient with type 1 diabetes arrives in the emergency department with a glucose level of 390 mg/dL and positive result for ketones. Vital signs are 110/70 mm Hg, 120 beats/minute, and 28 deep, sighing respirations/minute. What is the priority need for the patient? A. Oxygen B. Intravenous (IV) fluids C. Albuterol (Ventolin) D. Metformin (Glucophage)

B. Intravenous (IV) fluids A patient in diabetic ketoacidosis (DKA) needs IV fluids and insulin to stop the tissue breakdown resulting in ketone bodies and acidosis. The initial goal is fluid and electrolyte balance. Kussmaul respirations indicate the body is attempting to compensate by blowing off the carbon dioxide, but it is ineffective as long as the body continues to break down the ketone bodies and remains in metabolic acidosis.

A diabetic patient has a new order for inhaled insulin (Exubera). Which information about the patient indicates that the nurse should contact the patient before administering the Exubera? a. The patient has a history of a recent myocardial infarction. b. The patient's blood glucose is 224 mg/dl. c. The patient uses a bronchodilator to treat emphysema. d. The patient's temperature is 101.4° F.

C Rationale: Exubera is not recommended for patients with emphysema. The other data do not indicate any contraindication to using Exubera. Cognitive Level: Application Text Reference: p. 1263 Nursing Process: Assessment NCLEX: Physiological Integrity

A diagnosis of hyperglycemic hyperosmolar nonketotic coma (HHNC) is made for a patient with type 2 diabetes who is brought to the emergency department in an unresponsive state. The nurse will anticipate the need to a. administer glargine (Lantus) insulin. b. initiate oxygen by nasal cannula. c. insert a large-bore IV catheter. d. give 50% dextrose as a bolus.

C Rationale: HHNC is initially treated with large volumes of IV fluids to correct hypovolemia. Regular insulin is administered, not a long-acting insulin. There is no indication that the patient requires oxygen. Dextrose solutions will increase the patient's blood glucose and would be contraindicated. Cognitive Level: Application Text Reference: p. 1281 Nursing Process: Planning NCLEX: Physiological Integrity

** The patient with HHS presented with a glucose level of 800 mg/dL and is started on IV fluids and insulin. What action do you anticipate when the patient's glucose reaches 250 mg/dL? A. Administer an intravenous (IV) solution with 5% dextrose. B. Administer sodium polystyrene sulfate (Kayexalate). C. Slow the IV infusion rate to 40 mL/hour. D. Assess cardiac monitoring for peaked T waves.

A. Administer an intravenous (IV) solution with 5% dextrose. When blood glucose levels fall to approximately 250 mg/dL, IV fluids containing glucose are administered to prevent hypoglycemia. Kayexalate (for cation exchange) is used in the treatment of hyperkalemia, which causes peaked T waves on cardiac monitoring. In HHS hypokalemia may result from insulin moving the potassium intracellularly. Fluid replacement remains a priority, but it is given with dextrose. The infusion rate of 40 mL/hour keeps the vein open, but it is not the appropriate replacement rate.

During a clinic visit 3 months following a diagnosis of type 2 diabetes, the patient reports following a reduced-calorie diet. The patient has not lost any weight and did not bring the glucose-monitoring record. The nurse will plan to obtain a(n) a. fasting blood glucose level. b. urine dipstick for glucose. c. glycosylated hemoglobin level. d. oral glucose tolerance test.

C Rationale: The glycosylated hemoglobin (Hb A1C) test shows the overall control of glucose over 90 to 120 days. A fasting blood level indicates only the glucose level at one time. Urine glucose testing is not an accurate reflection of blood glucose level and does not reflect the glucose over a prolonged time. Oral glucose tolerance testing is done to diagnose diabetes, but is not used for monitoring glucose control once diabetes has been diagnosed. Cognitive Level: Application Text Reference: pp. 1258-1259 Nursing Process: Planning NCLEX: Physiological Integrity

Which lower extremity or foot finding is a sign of sensory neuropathy in a patient with diabetes mellitus? A. Dusky when legs are dependent B. Pitting pedal edema C. Intermittent claudication D. Strong pedal pulse

C. Intermittent claudication Peripheral arterial disease (PAD) is caused by a reduction of blood flow to the lower extremities. Classic signs include intermittent claudication, pain at rest, cold feet, loss of hair, delayed capillary filling, and dependent rubor. Dusky legs when they are dependent, pitting pedal edema, and a strong pedal pulse are signs of peripheral venous disease.

Which symptoms reported by a patient with diabetes mellitus are most important to follow-up? A. "My vision has been getting fuzzier over the past year." B. "I cannot read the small print anymore." C. "There is something like a veil of blackness coming across my vision." D. "I have yellow discharge from one eye."

C. "There is something like a veil of blackness coming across my vision." Diabetic retinopathy, particularly proliferative retinopathy, can cause retinal detachment, which has the classic new symptom of a veil coming across the field of vision. This requires emergency treatment. Chronic blurry vision can be cataracts and is not emergent. Change in the ability to read things near to the eye (presbyopia or farsightedness) is an age-related change and not emergent. Conjunctivitis needs treatment but is not as emergent as retinal detachment.

The patient has type 1 diabetes mellitus and is found unresponsive with cool and clammy skin. What action is a priority? A. Obtain a serum glucose level. B. Give hard candy under the tongue. C. Administer glucagon per standing order. D. Notify the health care provider.

C. Administer glucagon per standing order. The patient has signs and symptoms of hypoglycemia for which treatment should be the priority. Glucagon stimulates a strong hepatic response to convert glycogen to glucose and therefore makes glucose rapidly available. Waiting for a serum result (up to an hour) is improper because brain cells continue to die from a lack of glucose. Nothing solid should be placed in the mouth when the patient has an altered level of consciousness and can aspirate. With obvious symptoms, emergent treatment takes priority over notifying the health care provider.

** A diabetic patient has a serum glucose level of 824 mg/dL (45.7 mmol/L) and is unresponsive. After assessment of the patient, you suspect DKA rather than HHS based on the finding of A. polyuria. B. severe dehydration. C. rapid, deep respirations. D. decreased serum potassium.

C. rapid, deep respirations. Rapid, deep respirations are Kussmaul's and are are the body's attempt to reverse metabolic acidosis through exhalation of excess carbon dioxide. Symptoms of DKA include manifestations of dehydration, such as poor skin turgor, dry mucous membranes, tachycardia, and orthostatic hypotension. Kussmaul respirations (rapid, deep breathing associated with dyspnea) are the body's attempt to reverse metabolic acidosis through exhalation of excess carbon dioxide. Acetone is detected on the breath as a sweet, fruity odor.

A patient with diabetes mellitus who has multiple infections every year needs a mitral valve replacement. What is the most important preoperative teaching the nurse should provide to prevent a cardiac infection postoperatively? a. Avoid sick people and wash hands. b. Obtain comprehensive dental care. c. Maintain hemoglobin A1 c below 7%. d. Coughing and deep breathing with splinting

b. Obtain comprehensive dental care. A person with diabetes is at high risk for postoperative infections. The most important preoperative teaching to prevent a postoperative infection in the heart is to have the patient obtain comprehensive dental care because the risk of septicemia and infective endocarditis increases with poor dental health. Avoiding sick people, hand washing, maintaining hemoglobin A1c below 7%, and coughing and deep breathing with splinting would be important for any type of surgery, but not the priority with mitral valve replacement for this patient.

Which assessment is the most sensitive indicator that the IV fluid administration may be too rapid when treating a patient with DKA and a history of renal disease? A. Pedal edema B. Tachypnea C. Urine output of 40 mL/hour D. Change in the level of consciousness

D. Change in the level of consciousness Too rapid fluid replacement can lead to hyponatremia and cerebral edema. Pedal edema is a later and relatively insignificant sign. In a bedridden patient, edema is more evident in the sacral area. The Kussmaul respirations are expected; crackles auscultated in the lungs are a more sensitive indicator. The desired urine output for adequate hydration is 30 to 60 mL/hr.

What is the best teaching for a patient who is newly diagnosed with diabetes mellitus type 2? A. Read a Snellen chart yearly. B. Be checked out for presbycusis. C. Notify the doctor if your vision has color distortion. D. See an ophthalmologist for a dilated eye examination yearly.

D. See an ophthalmologist for a dilated eye examination yearly. The earliest and most treatable stages of diabetic retinopathy often produce no changes in the vision. Because of this, the patient with type 2 diabetes should have a dilated eye examination by an ophthalmologist at the time of diagnosis and annually thereafter for early detection and treatment.

What is most helpful in the prevention of nephropathy in a patient with diabetes mellitus? A. Acid-ash diet B. Ensuring adequate fluid intake for renal perfusion C. Preventing obstruction from benign prostatic hyperplasia (BPH) D. Stopping smoking

D. Stopping smoking Risk factors for diabetic nephropathy include hypertension, genetic predisposition, smoking, and chronic hyperglycemia. Diabetic nephropathy is an intrarenal microvascular complication in which the glomeruli of the kidney are damaged. The kidney receives about 25% of the cardiac output, and inadequate fluids or shock resulting in adequate perfusion is a prerenal cause. BPH is a postrenal cause of kidney pathology.

A diabetic patient who has reported burning foot pain at night receives a new prescription. Which information should the nurse teach the patient about amitriptyline (Elavil)? a. Amitriptyline decreases the depression caused by your foot pain. b. Amitriptyline helps prevent transmission of pain impulses to the brain. c. Amitriptyline corrects some of the blood vessel changes that cause pain. d. Amitriptyline improves sleep and makes you less aware of nighttime pain.

ANS: B Tricyclic antidepressants decrease the transmission of pain impulses to the spinal cord and brain. Tricyclic antidepressants also improve sleep quality and are used for depression, but that is not the major purpose for their use in diabetic neuropathy. The blood vessel changes that contribute to neuropathy are not affected by tricyclic antidepressants.

What is the priority action for the nurse to take if the patient with type 2 diabetes complains of blurred vision and irritability? a. call the physician b. administer insulin as ordered c. check the patient's blood glucose level d. assess for other neurologic symptoms.

c. check the patient's blood glucose level

Which statement would be correct for a patient with type 2 diabetes who was admitted to the hospital with pneumonia? a. the patient must receive insulin therapy to prevent ketoacidosis b. the patient has islet cell antibodies that have destroyed the pancreas's ability to produce insulin c. the patient has minimal or absent endogenous insulin secretion and requires daily insulin injections d. the patient may have sufficient endogenous insulin to prevent ketosis but is at risk for hyperosmolar hyperglycemic syndrome

d. the patient may have sufficient endogenous insulin to prevent ketosis but is at risk for hyperosmolar hyperglycemic syndrome

Laboratory results have been obtained for a 50-year-old patient with a 15-year history of type 2 diabetes. Which result reflects the expected pattern accompanying macrovascular disease as a complication of diabetes? a. Increased triglyceride levels b. Increased high-density lipoproteins (HDL) c. Decreased low-density lipoproteins (LDL) d. Decreased very-low-density lipoproteins (VLDL)

a. Increased triglyceride levels Macrovascular complications of diabetes include changes to large- and medium-sized blood vessels. They include cerebrovascular, cardiovascular, and peripheral vascular disease. Increased triglyceride levels are associated with these macrovascular changes. Increased HDL, decreased LDL, and decreased VLDL are positive in relation to atherosclerosis development.

The nurse is reviewing laboratory results for the clinic patients to be seen today. Which patient meets the diagnostic criteria for diabetes mellitus? a. A 48-year-old woman with a hemoglobin A1C of 8.4% b. A 58-year-old man with a fasting blood glucose of 111 mg/dL c. A 68-year-old woman with a random plasma glucose of 190 mg/dL d. A 78-year-old man with a 2-hour glucose tolerance plasma glucose of 184 mg/dL

a. A 48-year-old woman with a hemoglobin A1C of 8.4% Criteria for a diagnosis of diabetes mellitus include a hemoglobin A1C ≥ 6.5%, fasting plasma glucose level =126 mg/dL, 2-hour plasma glucose level =200 mg/dL during an oral glucose tolerance test, or classic symptoms of hyperglycemia or hyperglycemic crisis with a random plasma glucose =200 mg/dL.

Analyze the following diagnostic findings for your patient with type 2 diabetes. Which result will need further assessment? a. A1C 9% b. BP 126/80 c. FBG 130 d. LDL 100mg

a. A1C 9%

The nurse is assisting a patient with newly diagnosed type 2 diabetes to learn dietary planning as part of the initial management of diabetes. The nurse would encourage the patient to limit intake of which foods to help reduce the percent of fat in the diet? a. Cheese b. Broccoli c. Chicken d. Oranges

a. Cheese Cheese is a product derived from animal sources and is higher in fat and calories than vegetables, fruit, and poultry. Excess fat in the diet is limited to help avoid macrovascular changes.

The nurse has been teaching a patient with diabetes mellitus how to perform self-monitoring of blood glucose (SMBG). During evaluation of the patient's technique, the nurse identifies a need for additional teaching when the patient does what? a. Chooses a puncture site in the center of the finger pad. b. Washes hands with soap and water to cleanse the site to be used. c. Warms the finger before puncturing the finger to obtain a drop of blood. d. Tells the nurse that the result of 110 mg/dL indicates good control of diabetes.

a. Chooses a puncture site in the center of the finger pad The patient should select a site on the sides of the fingertips, not on the center of the finger pad as this area contains many nerve endings and would be unnecessarily painful. Washing hands, warming the finger, and knowing the results that indicate good control all show understanding of the teaching.

Which are appropriate therapies for patients with diabetes mellitus? a. Use of statins to treat dyslipidemia b. use of diuretics to treat nephropathy c.Use of ACE inhibitors to treat nephropathy d. use of serotonin agonists to decrease appetite. e. Use of laser photocoagulation to treat retinopathy

a. Use of statins to treat dyslipidemia c.Use of ACE inhibitors to treat nephropathy e. Use of laser photocoagulation to treat retinopathy

You are caring for a patient with newly diagnosed type 1 diabetes. What information is essential to include in your patient teaching before discharge from the hospital? (select all that apply) a. insulin administration b. elimination of sugar from diet c. need to reduce physical activity d. use of portable blood glucose monitor e. hypoglycemia prevention, symptoms, and treatment

a. insulin administration d. use of portable blood glucose monitor e. hypoglycemia prevention, symptoms, and treatment

Which patient with type 1 diabetes mellitus would be at the highest risk for developing hypoglycemic unawareness? a. A 58-year-old patient with diabetic retinopathy b. A 73-year-old patient who takes propranolol (Inderal) c. A 19-year-old patient who is on the school track team d. A 24-year-old patient with a hemoglobin A1C of 8.9%

b. A 73-year-old patient who takes propranolol (Inderal) Hypoglycemic unawareness is a condition in which a person does not experience the warning signs and symptoms of hypoglycemia until the person becomes incoherent and combative or loses consciousness. Hypoglycemic awareness is related to autonomic neuropathy of diabetes that interferes with the secretion of counterregulatory hormones that produce these symptoms. Older patients and patients who use â-adrenergic blockers (e.g., propranolol) are at risk for hypoglycemic unawareness.

The nurse is beginning to teach a diabetic patient about vascular complications of diabetes. What information is appropriate for the nurse to include? a. Macroangiopathy does not occur in type 1 diabetes but rather in type 2 diabetics who have severe disease. b. Microangiopathy is specific to diabetes and most commonly affects the capillary membranes of the eyes, kidneys, and skin. c. Renal damage resulting from changes in large- and medium-sized blood vessels can be prevented by careful glucose control. d. Macroangiopathy causes slowed gastric emptying and the sexual impotency experienced by a majority of patients with diabetes.

b. Microangiopathy is specific to diabetes and most commonly affects the capillary membranes of the eyes, kidneys, and skin. Microangiopathy occurs in diabetes mellitus. When it affects the eyes, it is called diabetic retinopathy. When the kidneys are affected, the patient has nephropathy. When the skin is affected, it can lead to diabetic foot ulcers. Macroangiopathy can occur in either type 1 or type 2 diabetes and contributes to cerebrovascular, cardiovascular, and peripheral vascular disease. Sexual impotency and slowed gastric emptying result from microangiopathy and neuropathy.

A 51-year-old patient with diabetes mellitus is scheduled for a fasting blood glucose level at 8:00 AM. The nurse instructs the patient to only drink water after what time? a. 6:00 PM on the evening before the test b. Midnight before the test c. 4:00 AM on the day of the test d. 7:00 AM on the day of the test

b. Midnight before the test Typically, a patient is ordered to be NPO for 8 hours before a fasting blood glucose level. For this reason, the patient who has a lab draw at 8:00 AM should not have any food or beverages containing any calories after midnight.

A patient screened for diabetes at a clinic has a fasting plasma glucose of 120 mg/dL (6.7 mmol/L). The nurse explains to the patient that this value: 1. Is diagnostic for diabetes. 2. Is normal, and diabetes is not a problem. 3. Reflects impaired glucose tolerance, which is an early stage of diabetes. 4. Indicates an intermediate stage between normal glucose use and diabetes.

3. Reflects impaired glucose tolerance, which is an early stage of diabetes. Impaired fasting glucose (fasting blood glucose level between 100 and 126 mg/dL) and impaired glucose tolerance (2-hour plasma glucose level between 140 and 199 mg/dL) represent an intermediate stage between normal glucose homeostasis and diabetes. This stage is called prediabetes

A patient with type 1 diabetes calls the clinic with complaints of nausea, vomiting, and diarrhea. It is most important that the nurse advise the patient to: 1. Hold the regular dose of insulin. 2. Drink cool fluids with high glucose content. 3. Check the blood glucose level every 2 to 4 hours. 4. Use a less strenuous form of exercise than usual until the illness resolves.

3. Check the blood glucose level every 2 to 4 hours. If a person with type 1 diabetes mellitus is ill, he or she should test blood glucose levels at least at 2-to-4-hour intervals to determine the effects of this stressor on the blood glucose level.

Ideally, the goal of patient diabetes education is to: 1. Make all patients responsible for the management of their disease. 2. Involve the patient's family and significant others in the care of the patient. 3. Enable the patient to become the most active participant in the management of the diabetes. 4. Provide the patient with as much information as soon as possible to prevent complications of diabetes.

3. Enable the patient to become the most active participant in the The goal of diabetes education is to enable the patient to become the most active participant in his or her own care.

The patient received regular insulin 10 units subcutaneously at 8:30 PM for a blood glucose level of 253 mg/dL. The nurse plans to monitor this patient for signs of hypoglycemia at which time related to the insulin's peak action? a. 8:40 PM to 9:00 PM b. 11:30 PM c. 10:30 PM to 1:30 AM d. 12:30 AM to 8:30 AM

c. 10:30 PM to 1:30 AM Regular insulin exerts peak action in 2 to 5 hours, making the patient most at risk for hypoglycemia between 10:30 PM and 1:30 AM. Rapid-acting insulin's onset is between 10-30 minutes with peak action and hypoglycemia most likely to occur between 9:00 PM and 11:30 PM. With intermediate acting insulin, hypoglycemia may occur from 12:30 AM to 8:30 AM.

A 63-year-old patient is newly diagnosed with type 2 diabetes. When developing an education plan, the nurse's first action should be to a. assess the patient's perception of what it means to have type 2 diabetes. b. demonstrate how to check glucose using capillary blood glucose monitoring. c. ask the patient's family to participate in the diabetes education program. d. discuss the need for the patient to actively participate in diabetes management.

A Rationale: Before planning education, the nurse should assess the patient's interest in and ability to self-manage the diabetes. After assessing the patient, the other nursing actions may be appropriate, but planning needs to be individualized to each patient. Cognitive Level: Application Text Reference: p.1264 Nursing Process: Planning NCLEX: Health Promotion and Maintenance

A patient with type 2 diabetes that is controlled with diet and metformin (Glucophage) also has severe rheumatoid arthritis (RA). During an acute exacerbation of the patient's arthritis, the health care provider prescribes prednisone (Deltasone) to control inflammation. The nurse will anticipate that the patient may a. require administration of insulin while taking prednisone. b. develop acute hypoglycemia during the RA exacerbation. c. have rashes caused by metformin-prednisone interactions. d. need a diet higher in calories while receiving prednisone.

A Rationale: Glucose levels increase when patients are taking corticosteroids, and insulin may be required to control blood glucose. Hypoglycemia is not a complication of RA exacerbation or prednisone use. Rashes are not an adverse effect caused by taking metformin and prednisone simultaneously. The patient is likely to have an increased appetite when taking prednisone, but it will be important to avoid weight gain for the patient with RA. Cognitive Level: Application Text Reference: pp. 1258, 1267 Nursing Process: Planning NCLEX: Physiological Integrity

After the home health nurse has taught a patient and family about how to use glargine and regular insulin safely, which action by the patient indicates that the teaching has been successful? a. The patient disposes of the open insulin vials after 4 weeks. b. The patient draws up the regular insulin in the syringe and then draws up the glargine. c. The patient stores extra vials of both types of insulin in the freezer until needed. d. The patient's family prefills the syringes weekly and stores them in the refrigerator.

A Rationale: Insulin can be stored at room temperature for 4 weeks. Glargine should not be mixed with other insulins or prefilled and stored. Freezing alters the insulin molecule and should not be done. Cognitive Level: Application Text Reference: p. 1261 Nursing Process: Evaluation NCLEX: Physiological Integrity

A patient using a split mixed-dose insulin regimen asks the nurse about the use of intensive insulin therapy to achieve tighter glucose control. The nurse should teach the patient that a. intensive insulin therapy requires three or more injections a day in addition to an injection of a basal long-acting insulin. b. intensive insulin therapy is indicated only for newly diagnosed type 1 diabetics who have never experienced ketoacidosis. c. studies have shown that intensive insulin therapy is most effective in preventing the macrovascular complications characteristic of type 2 diabetes. d. an insulin pump provides the best glucose control and requires about the same amount of attention as intensive insulin therapy.

A Rationale: Patients using intensive insulin therapy must check their glucose level four to six times daily and administer insulin accordingly. A previous episode of ketoacidosis is not a contraindication for intensive insulin therapy. Intensive insulin therapy is not confined to type 2 diabetics and would prevent microvascular changes as well as macrovascular changes. Intensive insulin therapy and an insulin pump are comparable in glucose control. Cognitive Level: Application Text Reference: p. 1263 Nursing Process: Implementation NCLEX: Physiological Integrity

A patient with type 1 diabetes has been using self-monitoring of blood glucose (SMBG) as part of diabetes management. During evaluation of the patient's technique of SMBG, the nurse identifies a need for additional teaching when the patient a. chooses a puncture site in the center of the finger pad. b. washes the puncture site using soap and water. c. says the result of 130 mg indicates good blood sugar control. d. hangs the arm down for a minute before puncturing the site.

A Rationale: The patient is taught to choose a puncture site at the side of the finger pad. The other patient actions indicate that teaching has been effective. Cognitive Level: Application Text Reference: p. 1270 Nursing Process: Evaluation NCLEX: Health Promotion and Maintenance

Which nursing action can the nurse delegate to unlicensed assistive personnel (UAP) who are working in the diabetic clinic? a. Measure the ankle-brachial index. b. Check for changes in skin pigmentation. c. Assess for unilateral or bilateral foot drop. d. Ask the patient about symptoms of depression.

ANS: A Checking systolic pressure at the ankle and brachial areas and calculating the ankle-brachial index is a procedure that can be done by UAP who have been trained in the procedure. The other assessments require more education and critical thinking and should be done by the registered nurse (RN).

** The elderly patient with type 2 diabetes mellitus presents to the clinic with a fever and productive cough. The diagnosis of pneumonia is made. You notice tenting skin, deep tongue furrows, and vital signs of 110/80 mm Hg, 120 beats/minute, and 24 breaths/minute. What assessment is important for you to obtain? A. Blood glucose B. Orthostatic blood pressures C. Urine ketones D. Temperature

A. Blood glucose HHS is typically seen in patients with type 2 diabetes and infection, such as pneumonia. The main presenting sign is a glucose level above 600 mg/dL. Enough evidence of dehydration already exists that orthostatic vital sign assessments are not a priority, and they are often inaccurate in the elderly due to poor vascular tone. Patients with HHS do not have elevated ketone levels, which is a key distinction between HHS and DKA. Temperature will eventually be taken but is often blunted in the elderly and diabetics. An infectious diagnosis has already been made. The glucose level for appropriate fluid and insulin treatment is the priority.

Laboratory results are available for a 54-year-old patient with a 15-year history of diabetes. Which result follows the expected pattern accompanying macrovascular disease as a complication of diabetes? A. Increased triglyceride levels B. Decreased low-density lipoprotein levels C. Increased high-density lipoprotein levels D. Decreased very-low-density lipoprotein levels

A. Increased triglyceride levels Macrovascular complications of diabetes include changes in medium- and large-sized blood vessels. They include cerebrovascular, cardiovascular, and peripheral vascular disease. Increased triglyceride levels are associated with these macrovascular changes. For this reason, the patient should limit the amount of fat in the diet.

What therapies are appropriate for patients with diabetes mellitus (select all that apply)? A. Use of statins to treat dyslipidemia B. Use of diuretics to treat nephropathy C. Use of angiotensin-converting enzyme (ACE) inhibitors to treat nephropathy D. Use of laser photocoagulation to treat retinopathy E. Use of protein restriction in patients with early signs of nephropathy

A. Use of statins to treat dyslipidemia C. Use of angiotensin-converting enzyme (ACE) inhibitors to treat nephropathy D. Use of laser photocoagulation to treat retinopathy In patients with diabetes who have microalbuminuria or macroalbuminuria, ACE inhibitors (-prils) or angiotensin II receptor antagonists (ARBs) (-sartans) should be used. Both of these drug classes are used to treat hypertension. The statin drugs are the most widely used lipid-lowering drugs. Laser photocoagulation therapy is indicated to reduce the risk of vision loss in patients with proliferative retinopathy, macular edema, and in some cases of nonproliferative retinopathy.

The patient presents to the emergency department with a glucose level of 400 mg/dL, ketone result of 2+, and rapid respirations with a fruity odor. What finding do you anticipate? A. pH below 7.30 B. Urine specific gravity below 1.005 C. High sodium bicarbonate levels D. Low blood urea nitrogen (BUN) level

A. pH below 7.30 The patient is in metabolic acidosis, which is a pH below 7.35. Dehydration results in a high urine specific gravity (at the upper end of the normal range, or above 1.025 to 1.030). Sodium bicarbonate levels are low in metabolic acidosis. The dehydration that occurs with DKA elevates the BUN level.

In which order will the nurse take these steps to prepare NPH 20 units and regular insulin 2 units using the same syringe? (Put a comma and a space between each answer choice [A, B, C, D, E]). a. Rotate NPH vial. b. Withdraw regular insulin. c. Withdraw 20 units of NPH. d. Inject 20 units of air into NPH vial. e. Inject 2 units of air into regular insulin vial.

ANS: A, D, E, B, C When mixing regular insulin with NPH, it is important to avoid contact between the regular insulin and the additives in the NPH that slow the onset, peak, and duration of activity in the longer-acting insulin.

Which question during the assessment of a diabetic patient will help the nurse identify autonomic neuropathy? a. "Do you feel bloated after eating?" b. "Have you seen any skin changes?" c. "Do you need to increase your insulin dosage when you are stressed?" d. "Have you noticed any painful new ulcerations or sores on your feet?"

ANS: A Autonomic neuropathy can cause delayed gastric emptying, which results in a bloated feeling for the patient. The other questions are also appropriate to ask but would not help in identifying autonomic neuropathy.

In order to assist an older diabetic patient to engage in moderate daily exercise, which action is most important for the nurse to take? a. Determine what type of activities the patient enjoys. b. Remind the patient that exercise will improve self-esteem. c. Teach the patient about the effects of exercise on glucose level. d. Give the patient a list of activities that are moderate in intensity.

ANS: A Because consistency with exercise is important, assessment for the types of exercise that the patient finds enjoyable is the most important action by the nurse in ensuring adherence to an exercise program. The other actions will also be implemented but are not the most important in improving compliance.

Which statement by the patient indicates a need for additional instruction in administering insulin? a. "I need to rotate injection sites among my arms, legs, and abdomen each day." b. "I can buy the 0.5 mL syringes because the line markings will be easier to see." c. "I should draw up the regular insulin first after injecting air into the NPH bottle." d. "I do not need to aspirate the plunger to check for blood before injecting insulin."

ANS: A Rotating sites is no longer recommended because there is more consistent insulin absorption when the same site is used consistently. The other patient statements are accurate and indicate that no additional instruction is needed

A female patient is scheduled for an oral glucose tolerance test. Which information from the patient's health history is most important for the nurse to communicate to the health care provider? a. The patient uses oral contraceptives. b. The patient runs several days a week. c. The patient has been pregnant three times. d. The patient has a family history of diabetes.

ANS: A Oral contraceptive use may falsely elevate oral glucose tolerance test (OGTT) values. Exercise and a family history of diabetes both can affect blood glucose but will not lead to misleading information from the OGTT. History of previous pregnancies may provide informational about gestational glucose tolerance, but will not lead to misleading information from the OGTT.

A 32-year-old patient with diabetes is starting on intensive insulin therapy. Which type of insulin will the nurse discuss using for mealtime coverage? a. Lispro (Humalog) b. Glargine (Lantus) c. Detemir (Levemir) d. NPH (Humulin N)

ANS: A Rapid- or short-acting insulin is used for mealtime coverage for patients receiving intensive insulin therapy. NPH, glargine, or detemir will be used as the basal insulin.

A 27-year-old patient admitted with diabetic ketoacidosis (DKA) has a serum glucose level of 732 mg/dL and serum potassium level of 3.1 mEq/L. Which action prescribed by the health care provider should the nurse take first? a. Place the patient on a cardiac monitor. b. Administer IV potassium supplements. c. Obtain urine glucose and ketone levels. d. Start an insulin infusion at 0.1 units/kg/hr.

ANS: A Hypokalemia can lead to potentially fatal dysrhythmias such as ventricular tachycardia and ventricular fibrillation, which would be detected with electrocardiogram (ECG) monitoring. Because potassium must be infused over at least 1 hour, the nurse should initiate cardiac monitoring before infusion of potassium. Insulin should not be administered without cardiac monitoring because insulin infusion will further decrease potassium levels. Urine glucose and ketone levels are not urgently needed to manage the patient's care.

Which patient action indicates a good understanding of the nurse's teaching about the use of an insulin pump? a. The patient programs the pump for an insulin bolus after eating. b. The patient changes the location of the insertion site every week. c. The patient takes the pump off at bedtime and starts it again each morning. d. The patient plans for a diet that is less flexible when using the insulin pump.

ANS: A In addition to the basal rate of insulin infusion, the patient will adjust the pump to administer a bolus after each meal, with the dosage depending on the oral intake. The insertion site should be changed every 2 or 3 days. There is more flexibility in diet and exercise when an insulin pump is used. The pump will deliver a basal insulin rate 24 hours a day.

A 38-year-old patient who has type 1 diabetes plans to swim laps daily at 1:00 PM. The clinic nurse will plan to teach the patient to a. check glucose level before, during, and after swimming. b. delay eating the noon meal until after the swimming class. c. increase the morning dose of neutral protamine Hagedorn (NPH) insulin. d. time the morning insulin injection so that the peak occurs while swimming.

ANS: A The change in exercise will affect blood glucose, and the patient will need to monitor glucose carefully to determine the need for changes in diet and insulin administration. Because exercise tends to decrease blood glucose, patients are advised to eat before exercising. Increasing the morning NPH or timing the insulin to peak during exercise may lead to hypoglycemia, especially with the increased exercise.

A patient receives aspart (NovoLog) insulin at 8:00 AM. Which time will it be most important for the nurse to monitor for symptoms of hypoglycemia? a. 10:00 AM b. 12:00 AM c. 2:00 PM d. 4:00 PM

ANS: A The rapid-acting insulins peak in 1 to 3 hours. The patient is not at a high risk for hypoglycemia at the other listed times, although hypoglycemia may occur.

A 55-year-old female patient with type 2 diabetes has a nursing diagnosis of imbalanced nutrition: more than body requirements. Which goal is most important for this patient? a. The patient will reach a glycosylated hemoglobin level of less than 7%. b. The patient will follow a diet and exercise plan that results in weight loss. c. The patient will choose a diet that distributes calories throughout the day. d. The patient will state the reasons for eliminating simple sugars in the diet.

ANS: A The complications of diabetes are related to elevated blood glucose, and the most important patient outcome is the reduction of glucose to near-normal levels. The other outcomes also are appropriate but are not as high in priority.

A 54-year-old patient is admitted with diabetic ketoacidosis. Which admission order should the nurse implement first? a. Infuse 1 liter of normal saline per hour. b. Give sodium bicarbonate 50 mEq IV push. c. Administer regular insulin 10 U by IV push. d. Start a regular insulin infusion at 0.1 units/kg/hr.

ANS: A The most urgent patient problem is the hypovolemia associated with diabetic ketoacidosis (DKA), and the priority is to infuse IV fluids. The other actions can be done after the infusion of normal saline is initiated.

After change-of-shift report, which patient should the nurse assess first? a. 19-year-old with type 1 diabetes who has a hemoglobin A1C of 12% b. 23-year-old with type 1 diabetes who has a blood glucose of 40 mg/dL c. 40-year-old who is pregnant and whose oral glucose tolerance test is 202 mg/dL d. 50-year-old who uses exenatide (Byetta) and is complaining of acute abdominal pain

ANS: B Because the brain requires glucose to function, untreated hypoglycemia can cause unconsciousness, seizures, and death. The nurse will rapidly assess and treat the patient with low blood glucose. The other patients also have symptoms that require assessments and/or interventions, but they are not at immediate risk for life-threatening complications.

Which action should the nurse take after a 36-year-old patient treated with intramuscular glucagon for hypoglycemia regains consciousness? a. Assess the patient for symptoms of hyperglycemia. b. Give the patient a snack of peanut butter and crackers. c. Have the patient drink a glass of orange juice or nonfat milk. d. Administer a continuous infusion of 5% dextrose for 24 hours.

ANS: B Rebound hypoglycemia can occur after glucagon administration, but having a meal containing complex carbohydrates plus protein and fat will help prevent hypoglycemia. Orange juice and nonfat milk will elevate blood glucose rapidly, but the cheese and crackers will stabilize blood glucose. Administration of IV glucose might be used in patients who were unable to take in nutrition orally. The patient should be assessed for symptoms of hypoglycemia after glucagon administration.

Which laboratory value reported to the nurse by the unlicensed assistive personnel (UAP) indicates the most urgent need for the nurse's assessment of the patient? a. Bedtime glucose of 140 mg/dL b. Noon blood glucose of 52 mg/dL c. Fasting blood glucose of 130 mg/dL d. 2-hr postprandial glucose of 220 mg/dL

ANS: B The nurse should assess the patient with a blood glucose level of 52 mg/dL for symptoms of hypoglycemia and give the patient a carbohydrate-containing beverage such as orange juice. The other values are within an acceptable range or not immediately dangerous for a diabetic patient.

The nurse identifies a need for additional teaching when the patient who is self-monitoring blood glucose a. washes the puncture site using warm water and soap. b. chooses a puncture site in the center of the finger pad. c. hangs the arm down for a minute before puncturing the site. d. says the result of 120 mg indicates good blood sugar control.

ANS: B The patient is taught to choose a puncture site at the side of the finger pad because there are fewer nerve endings along the side of the finger pad. The other patient actions indicate that teaching has been effective.

Which information will the nurse include when teaching a 50-year-old patient who has type 2 diabetes about glyburide (Micronase, DiaBeta, Glynase)? a. Glyburide decreases glucagon secretion from the pancreas. b. Glyburide stimulates insulin production and release from the pancreas. c. Glyburide should be taken even if the morning blood glucose level is low. d. Glyburide should not be used for 48 hours after receiving IV contrast media.

ANS: B The sulfonylureas stimulate the production and release of insulin from the pancreas. If the glucose level is low, the patient should contact the health care provider before taking the glyburide, because hypoglycemia can occur with this class of medication. Metformin should be held for 48 hours after administration of IV contrast media, but this is not necessary for glyburide. Glucagon secretion is not affected by glyburide.

An active 28-year-old male with type 1 diabetes is being seen in the endocrine clinic. Which finding may indicate the need for a change in therapy? a. Hemoglobin A1C level 6.2% b. Blood pressure 146/88 mmHg c. Heart rate at rest 58 beats/minute d. High density lipoprotein (HDL) level 65 mg/dL

ANS: B To decrease the incidence of macrovascular and microvascular problems in patients with diabetes, the goal blood pressure is usually 130/80. An A1C less than 6.5%, a low resting heart rate (consistent with regular aerobic exercise in a young adult), and an HDL level of 65 mg/dL all indicate that the patient's diabetes and risk factors for vascular disease are well controlled.

Which patient action indicates good understanding of the nurse's teaching about administration of aspart (NovoLog) insulin? a. The patient avoids injecting the insulin into the upper abdominal area. b. The patient cleans the skin with soap and water before insulin administration. c. The patient stores the insulin in the freezer after administering the prescribed dose. d. The patient pushes the plunger down while removing the syringe from the injection site.

ANS: B Cleaning the skin with soap and water or with alcohol is acceptable. Insulin should not be frozen. The patient should leave the syringe in place for about 5 seconds after injection to be sure that all the insulin has been injected. The upper abdominal area is one of the preferred areas for insulin injection.

An unresponsive patient with type 2 diabetes is brought to the emergency department and diagnosed with hyperosmolar hyperglycemic syndrome (HHS). The nurse will anticipate the need to a. give a bolus of 50% dextrose. b. insert a large-bore IV catheter. c. initiate oxygen by nasal cannula. d. administer glargine (Lantus) insulin.

ANS: B HHS is initially treated with large volumes of IV fluids to correct hypovolemia. Regular insulin is administered, not a long-acting insulin. There is no indication that the patient requires oxygen. Dextrose solutions will increase the patient's blood glucose and would be contraindicated.

To monitor for complications in a patient with type 2 diabetes, which tests will the nurse in the diabetic clinic schedule at least annually (select all that apply)? a. Chest x-ray b. Blood pressure c. Serum creatinine d. Urine for microalbuminuria e. Complete blood count (CBC) f. Monofilament testing of the foot

ANS: B, C, D, F Blood pressure, serum creatinine, urine testing for microalbuminuria, and monofilament testing of the foot are recommended at least annually to screen for possible microvascular and macrovascular complications of diabetes. Chest x-ray and CBC might be ordered if the diabetic patient presents with symptoms of respiratory or infectious problems but are not routinely included in screening.

Cardiac monitoring is initiated for a patient in diabetic ketoacidosis. The nurse recognizes that this measure is important to identify: 1. Dysrhythmias resulting from hypokalemia. 2. Fluid overload resulting from aggressive fluid replacement. 3. The presence of hypovolemic shock related to osmotic diuresis. 4. Cardiovascular collapse resulting from the effects of excess glucose on cardiac cells.

1. Dysrhythmias resulting from hypokalemia. Electrolytes are depleted in diabetic ketoacidosis. Osmotic diuresis occurs with depletion of sodium, potassium, chloride, magnesium, and phosphate. Hypokalemia may lead to ventricular dysrhythmias such as premature ventricular complexes and bradycardia.

Case Study: 52-year-old woman was diagnosed with type 2 diabetes 6 years ago. Did not follow up with recommendations for care Complaining of weakness in her right foot Began 1 month ago Difficult to dorsiflex and feels numb Also complains of an itching rash in her groin area Has had rash on and off for many years Worse when weather is warm Increased thirst and frequent nighttime urination Denies other weakness, numbness, changes in vision She works as a banking executive and gets little exercise. She has gained 18 pounds over the past year and eats a high-fat diet. BP 162/98 Sensory exam to light touch, proprioception, and vibration slightly diminished on both feet Erythematous scaling rash in both inguinal areas and in axillae Random glucose test 253 mg/dL Hb A1C 9.1% Urine dipstick positive for glucose and negative for protein Wet prep of smear from rash consistent with Candida albicans ECG with evidence of early ventricular hypertrophy by voltage ------------------------------------ 1.She wants to know why all of these changes have been happening to her body. How would you explain this to her? 2. What is the priority nursing intervention? 3.What teaching should be done with her?

1. Her lack of glucose control has affected multiple other parts of her body, including the nerves in her lower extremities, cardiovascular function, and ability to fight off infection. 2. Treatment of hyperglycemia, neuropathy, and hypertension, and, more important, patient teaching. 3. Stress the importance of adhering to her medical regimen and monitoring her blood glucose regularly. Teach her the repercussions if she does not do these things.

Intramuscular glucagon is administered to an unresponsive patient for treatment of hypoglycemia. Which action should the nurse take after the patient regains consciousness? a. Give the patient a snack of cheese and crackers. b. Have the patient drink a glass of orange juice or nonfat milk. c. Administer a continuous infusion of 5% dextrose for 24 hours. d. Assess the patient for symptoms of hyperglycemia.

A Rationale: Rebound hypoglycemia can occur after glucagon administration, but having a meal containing complex carbohydrates plus protein and fat will help prevent hypoglycemia. Orange juice and nonfat milk will elevate blood sugar rapidly, but the cheese and crackers will stabilize blood sugar. Administration of glucose intravenously might be used in patients who were unable to take in nutrition orally. The patient should be assessed for symptoms of hypoglycemia after glucagon administration. Cognitive Level: Application Text Reference: p. 1282 Nursing Process: Implementation NCLEX: Physiological Integrity

The nurse has been teaching the patient to administer a dose of 10 units of regular insulin and 28 units of NPH insulin. The statement by the patient that indicates a need for additional instruction is, a. "I need to rotate injection sites among my arms, legs, and abdomen each day." b. "I will buy the 0.5-ml syringes because the line markings will be easier to see." c. "I should draw up the regular insulin first after injecting air into the NPH bottle." d. "I do not need to aspirate the plunger to check for blood before I inject the insulin."

A Rationale: Rotating sites is no longer necessary because all insulin is now purified human insulin, and the risk for lipodystrophy is low. The other patient statements are accurate and indicate that no additional instruction is needed. Cognitive Level: Application Text Reference: p. 1262 Nursing Process: Evaluation NCLEX: Health Promotion and Maintenance

Cardiac monitoring is initiated for a patient in diabetic ketoacidosis (DKA). The nurse recognizes that this measure is important to identify a. electrocardiographic (ECG) changes and dysrhythmias related to hypokalemia. b. fluid overload resulting from aggressive fluid replacement. c. the presence of hypovolemic shock related to osmotic diuresis. d. cardiovascular collapse resulting from the effects of hyperglycemia.

A Rationale: The hypokalemia associated with metabolic acidosis can lead to potentially fatal dysrhythmias such as ventricular tachycardia and ventricular fibrillation, which would be detected with ECG monitoring. Fluid overload, hypovolemia, and cardiovascular collapse are possible complications of DKA, but cardiac monitoring would not detect theses. Cognitive Level: Application Text Reference: p. 1281 Nursing Process: Assessment NCLEX: Physiological Integrity

When teaching a patient with type 2 diabetes about taking glipizide (Glucotrol), the nurse determines that additional teaching about the medication is needed when the patient says, a. "Since I can take oral drugs rather than insulin, my diabetes is not serious and won't cause many complications." b. "If I overeat at a meal, I will still take just the usual dose of medication." c. "If I become ill, I may have to take insulin to control my blood sugar." d. "I should check with my doctor before taking any other medications because there are many that will affect glucose levels."

A Rationale: The patient should understand that type 2 diabetes places the patient at risk for many complications and that good glucose control is as important when taking oral agents as when using insulin. The other statements are accurate and indicate good understanding of the use of glipizide. Cognitive Level: Application Text Reference: p. 1275 Nursing Process: Evaluation NCLEX: Health Promotion and Maintenance

While hospitalized and recovering from an episode of diabetic ketoacidosis, the patient calls the nurse and reports feeling anxious, nervous, and sweaty. Based on the patient's report, the nurse should a. obtain a glucose reading using a finger stick. b. administer 1 mg glucagon subcutaneously. c. have the patient eat a candy bar. d. have the patient drink 4 ounces of orange juice.

A Rationale: The patient's clinical manifestations are consistent with hypoglycemia and the initial action should be to check the patient's glucose with a finger stick or order a stat blood glucose. If the glucose is low, the patient should ingest a rapid-acting carbohydrate, such as orange juice. Glucagon might be given if the patient's symptoms become worse or if the patient is unconscious. Candy bars contain fat, which would slow down the absorption of sugar and delay the response to treatment. Cognitive Level: Application Text Reference: p. 1282 Nursing Process: Implementation NCLEX: Physiological Integrity

Glyburide (Micronase, DiaBeta, Glynase) is prescribed for a patient whose type 2 diabetes has not been controlled with diet and exercise. When teaching the patient about glyburide, the nurse explains that a. glyburide stimulates insulin production and release from the pancreas. b. the patient should not take glyburide for 48 hours after receiving IV contrast media. c. glyburide should be taken even when the blood glucose level is low in the morning. d. glyburide decreases glucagon secretion.

A Rationale: The sulfonylureas stimulate the production and release of insulin from the pancreas. If the glucose level is low, the patient should contact the health care provider before taking the glyburide, since hypoglycemia can occur with this category of medication. Metformin should be held for 48 hours after administration of IV contract, but this is not necessary for glyburide. Cognitive Level: Application Text Reference: pp. 1265-1266 Nursing Process: Implementation NCLEX: Physiological Integrity

The patient with diabetes and shortness of breath is brought from the nursing home to the hospital emergency department. The electrocardiogram (ECG) shows evidence of a myocardial infarction (MI), but the patient denied ever having chest pain. Which is the best explanation of what happened? A. The patient had a "silent" MI related to autonomic neuropathy. B. The patient had chest pain but forgot because of dementia. C. The patient minimized the chest pain because he was worried about costs. D. The patient has the psychologic defense mechanism of denial.

A. The patient had a "silent" MI related to autonomic neuropathy. Cardiovascular abnormalities associated with autonomic neuropathy include painless myocardial infarction. Shortness of breath related to decreased cardiac functioning can be the first overt sign or symptom.

The nurse has administered 4 oz of orange juice to an alert patient whose blood glucose was 62 mg/dL. Fifteen minutes later, the blood glucose is 67 mg/dL. Which action should the nurse take next? a. Give the patient 4 to 6 oz more orange juice. b. Administer the PRN glucagon (Glucagon) 1 mg IM. c. Have the patient eat some peanut butter with crackers. d. Notify the health care provider about the hypoglycemia.

ANS: A The "rule of 15" indicates that administration of quickly acting carbohydrates should be done 2 to 3 times for a conscious patient whose glucose remains less than 70 mg/dL before notifying the health care provider. More complex carbohydrates and fats may be used once the glucose has stabilized. Glucagon should be used if the patient's level of consciousness decreases so that oral carbohydrates can no longer be given.

Which information will the nurse include in teaching a female patient who has peripheral arterial disease, type 2 diabetes, and sensory neuropathy of the feet and legs? a. Choose flat-soled leather shoes. b. Set heating pads on a low temperature. c. Use callus remover for corns or calluses. d. Soak feet in warm water for an hour each day.

ANS: A The patient is taught to avoid high heels and that leather shoes are preferred. The feet should be washed, but not soaked, in warm water daily. Heating pad use should be avoided. Commercial callus and corn removers should be avoided. The patient should see a specialist to treat these problems

A 34-year-old has a new diagnosis of type 2 diabetes. The nurse will discuss the need to schedule a dilated eye exam a. every 2 years. b. as soon as possible. c. when the patient is 39 years old. d. within the first year after diagnosis.

ANS: B Because many patients have some diabetic retinopathy when they are first diagnosed with type 2 diabetes, a dilated eye exam is recommended at the time of diagnosis and annually thereafter. Patients with type 1 diabetes should have dilated eye exams starting 5 years after they are diagnosed and then annually.

The nurse is preparing to teach a 43-year-old man who is newly diagnosed with type 2 diabetes about home management of the disease. Which action should the nurse take first? a. Ask the patient's family to participate in the diabetes education program. b. Assess the patient's perception of what it means to have diabetes mellitus. c. Demonstrate how to check glucose using capillary blood glucose monitoring. d. Discuss the need for the patient to actively participate in diabetes management.

ANS: B Before planning teaching, the nurse should assess the patient's interest in and ability to self-manage the diabetes. After assessing the patient, the other nursing actions may be appropriate, but planning needs to be individualized to each patient.

The nurse is taking a health history from a 29-year-old pregnant patient at the first prenatal visit. The patient reports no personal history of diabetes but has a parent who is diabetic. Which action will the nurse plan to take first? a. Teach the patient about administering regular insulin. b. Schedule the patient for a fasting blood glucose level. c. Discuss an oral glucose tolerance test for the twenty-fourth week of pregnancy. d. Provide teaching about an increased risk for fetal problems with gestational diabetes.

ANS: B Patients at high risk for gestational diabetes should be screened for diabetes on the initial prenatal visit. An oral glucose tolerance test may also be used to check for diabetes, but it would be done before the twenty-fourth week. The other actions may also be needed (depending on whether the patient develops gestational diabetes), but they are not the first actions that the nurse should take.

When a patient who takes metformin (Glucophage) to manage type 2 diabetes develops an allergic rash from an unknown cause, the health care provider prescribes prednisone (Deltasone). The nurse will anticipate that the patient may a. need a diet higher in calories while receiving prednisone. b. develop acute hypoglycemia while taking the prednisone. c. require administration of insulin while taking prednisone. d. have rashes caused by metformin-prednisone interactions.

ANS: C Glucose levels increase when patients are taking corticosteroids, and insulin may be required to control blood glucose. Hypoglycemia is not a side effect of prednisone. Rashes are not an adverse effect caused by taking metformin and prednisone simultaneously. The patient may have an increased appetite when taking prednisone, but will not need a diet that is higher in calories.

A 26-year-old female with type 1 diabetes develops a sore throat and runny nose after caring for her sick toddler. The patient calls the clinic for advice about her symptoms and a blood glucose level of 210 mg/dL despite taking her usual glargine (Lantus) and lispro (Humalog) insulin. The nurse advises the patient to a. use only the lispro insulin until the symptoms are resolved. b. limit intake of calories until the glucose is less than 120 mg/dL. c. monitor blood glucose every 4 hours and notify the clinic if it continues to rise. d. decrease intake of carbohydrates until glycosylated hemoglobin is less than 7%.

ANS: C Infection and other stressors increase blood glucose levels and the patient will need to test blood glucose frequently, treat elevations appropriately with lispro insulin, and call the health care provider if glucose levels continue to be elevated. Discontinuing the glargine will contribute to hyperglycemia and may lead to diabetic ketoacidosis (DKA). Decreasing carbohydrate or caloric intake is not appropriate because the patient will need more calories when ill. Glycosylated hemoglobin testing is not used to evaluate short-term alterations in blood glucose.

The nurse determines a need for additional instruction when the patient with newly diagnosed type 1 diabetes says which of the following? a. "I can have an occasional alcoholic drink if I include it in my meal plan." b. "I will need a bedtime snack because I take an evening dose of NPH insulin." c. "I can choose any foods, as long as I use enough insulin to cover the calories." d. "I will eat something at meal times to prevent hypoglycemia, even if I am not hungry."

ANS: C Most patients with type 1 diabetes need to plan diet choices very carefully. Patients who are using intensified insulin therapy have considerable flexibility in diet choices but still should restrict dietary intake of items such as fat, protein, and alcohol. The other patient statements are correct and indicate good understanding of the diet instruction.

ANS: C Since exenatide slows gastric emptying, oral medications should be taken at least an hour before the exenatide to avoid slowing absorption. Exenatide is injected and increases feelings of satiety. Hypoglycemia can occur with this medication.

ANS: C The decrease in renal function may indicate a need to adjust the dose of metformin or change to a different medication. In older patients, the goal for A1C may be higher in order to avoid complications associated with hypoglycemia. The nurse will plan on scheduling the patient for an eye exam and addressing the questions about diet, but the biggest concern is the patient's decreased renal function.

A 48-year-old male patient screened for diabetes at a clinic has a fasting plasma glucose level of 120 mg/dL (6.7 mmol/L). The nurse will plan to teach the patient about a. self-monitoring of blood glucose. b. using low doses of regular insulin. c. lifestyle changes to lower blood glucose. d. effects of oral hypoglycemic medications.

ANS: C The patient's impaired fasting glucose indicates prediabetes, and the patient should be counseled about lifestyle changes to prevent the development of type 2 diabetes. The patient with prediabetes does not require insulin or oral hypoglycemics for glucose control and does not need to self-monitor blood glucose.

A hospitalized diabetic patient received 38 U of NPH insulin at 7:00 AM. At 1:00 PM, the patient has been away from the nursing unit for 2 hours, missing the lunch delivery while awaiting a chest x-ray. To prevent hypoglycemia, the best action by the nurse is to a. save the lunch tray for the patient's later return to the unit. b. ask that diagnostic testing area staff to start a 5% dextrose IV. c. send a glass of milk or orange juice to the patient in the diagnostic testing area. d. request that if testing is further delayed, the patient be returned to the unit to eat.

ANS: D Consistency for mealtimes assists with regulation of blood glucose, so the best option is for the patient to have lunch at the usual time. Waiting to eat until after the procedure is likely to cause hypoglycemia. Administration of an IV solution is unnecessarily invasive for the patient. A glass of milk or juice will keep the patient from becoming hypoglycemic but will cause a rapid rise in blood glucose because of the rapid absorption of the simple carbohydrate in these items.

A 1200-calorie diet and exercise are prescribed for a patient with newly diagnosed type 2 diabetes. The patient tells the nurse, "I hate to exercise! Can't I just follow the diet to keep my glucose under control?" The nurse teaches the patient that the major purpose of exercise for diabetics is to a. increase energy and sense of well-being, which will help with body image. b. facilitate weight loss, which will decrease peripheral insulin resistance. c. improve cardiovascular endurance, which is important for diabetics. d. set a successful pattern, which will help in making other needed changes.

B Rationale: Exercise is essential to decrease insulin resistance and improve blood glucose control. Increased energy, improved cardiovascular endurance, and setting a pattern of success are secondary benefits of exercise, but they are not the major reason. Cognitive Level: Application Text Reference: p. 1269 Nursing Process: Implementation NCLEX: Physiological Integrity

A patient with type 1 diabetes has an unusually high morning glucose measurement, and the health care provider wants the patient evaluated for possible Somogyi effect. The nurse will plan to a. administer an increased dose of NPH insulin in the evening. b. obtain the patient's blood glucose at 3:00 in the morning. c. withhold the nighttime snack and check the glucose at 6:00 AM. d. check the patient for symptoms of hypoglycemia at 2:00 to 4:00 AM.

B Rationale: In the Somogyi effect, the patient's blood glucose drops in the early morning hours (in response to excess insulin administration), which causes the release of hormones that result in a rebound hyperglycemia. It is important to check the blood glucose in the early morning hours to detect the initial hypoglycemia. An increased evening NPH dose or holding the nighttime snack will further increase the risk for early morning hypoglycemia. Information about symptoms of hypoglycemia will not be as accurate as checking the patient's blood glucose in determining whether the patient has the Somogyi effect. Cognitive Level: Application Text Reference: pp. 1263-1264 Nursing Process: Planning NCLEX: Physiological Integrity

A patient with type 1 diabetes who uses glargine (Lantus) and lispro (Humalog) insulin develops a sore throat, cough, and fever. When the patient calls the clinic to report the symptoms and a blood glucose level of 210 mg/dl, the nurse advises the patient to a. use only the lispro insulin until the symptoms of infection are resolved. b. monitor blood glucose every 4 hours and notify the clinic if it continues to rise. c. decrease intake of carbohydrates until glycosylated hemoglobin is less than 7%. d. limit intake to non-calorie-containing liquids until the glucose is within the usual range.

B Rationale: Infection and other stressors increase blood glucose levels and the patient will need to test blood glucose frequently, treat elevations appropriately with insulin, and call the health care provider if glucose levels continue to be elevated. Discontinuing the glargine will contribute to hyperglycemia and may lead to DKA. Decreasing carbohydrate or caloric intake is not appropriate as the patient will need more calories when ill. Glycosylated hemoglobins are not used to test for short-term alterations in blood glucose. Cognitive Level: Application Text Reference: p. 1272 Nursing Process: Implementation NCLEX: Health Promotion and Maintenance

During a diabetes screening program, a patient tells the nurse, "My mother died of complications of type 2 diabetes. Can I inherit diabetes?" The nurse explains that a. as long as the patient maintains normal weight and exercises, type 2 diabetes can be prevented. b. the patient is at a higher than normal risk for type 2 diabetes and should have periodic blood glucose level testing. c. there is a greater risk for children developing type 2 diabetes when the father has type 2 diabetes. d. although there is a tendency for children of people with type 2 diabetes to develop diabetes, the risk is higher for those with type 1 diabetes.

B Rationale: Offspring of people with type 2 diabetes are at higher risk for developing type 2 diabetes. The risk can be decreased, but not prevented, by maintenance of normal weight and exercising. The risk for children of a person with type 1 diabetes to develop diabetes is higher when it is the father who has the disease. Offspring of people with type 2 diabetes are more likely to develop diabetes than offspring of those with type 1 diabetes. Cognitive Level: Application Text Reference: p. 1256 Nursing Process: Implementation NCLEX: Physiological Integrity

The patient is managed with NPH and regular insulin injections before breakfast and before dinner. When is the patient most likely to have a hypoglycemic reaction? A. After breakfast B. Before lunch C. During lunch D. After lunch

B. Before lunch The regular insulin peak occurs about 2 to 3 hours with a duration of 5 to 6 hours. If too much insulin or not enough food is given, the most likely time of hypoglycemia is before lunch, when the regular insulin is still present, the NPH has its onset, and the breakfast food has been metabolized.

You are beginning to teach a diabetic patient about the vascular complications of diabetes. Which information is appropriate for you to include? A. Macroangiopathy does not occur in type 1 diabetes but does affect type 2 diabetics who have severe disease. B. Microangiopathy is specific to diabetes and most commonly affects the capillary membranes of the eyes, kidneys, and skin. C. Renal damage resulting from changes in large and medium-sized blood vessels can be prevented by careful glucose control. D. Macroangiopathy causes slowed gastric emptying and the sexual impotency experienced by most patients with diabetes.

B. Microangiopathy is specific to diabetes and most commonly affects the capillary membranes of the eyes, kidneys, and skin. Microangiopathy occurs in type 1 and type 2 diabetes mellitus. When it affects the eyes, it is called diabetic retinopathy. When kidneys are affected, the patient has nephropathy. When the skin is affected, it can lead to diabetic foot ulcers. Sexual impotency and slowed gastric emptying result from microangiopathy.

A type 1 diabetic patient who was admitted with severe hypoglycemia and treated tells the nurse, "I did not have any of the usual symptoms of hypoglycemia." Which question by the nurse will help identify a possible reason for the patient's hypoglycemic unawareness? a. "Do you use any calcium-channel blocking drugs for blood pressure?" b. "Have you observed any recent skin changes?" c. "Do you notice any bloating feeling after eating?" d. "Have you noticed any painful new ulcerations or sores on your feet?"

C Rationale: Hypoglycemic unawareness is caused by autonomic neuropathy, which would also cause delayed gastric emptying. Calcium-channel blockers are not associated with hypoglycemic unawareness, although -adrenergic blockers can prevent patients from having symptoms of hypoglycemia. Skin changes can occur with diabetes, but these are not associated with autonomic neuropathy. If the patient can feel painful areas on the feet, neuropathy has not occurred. Cognitive Level: Application Text Reference: p. 1281 Nursing Process: Assessment NCLEX: Physiological Integrity

A patient who has just been diagnosed with type 2 diabetes is 5 ft 4 in (160 cm) tall and weighs 182 pounds (82 kg). A nursing diagnosis of imbalanced nutrition: more than body requirements is developed. Which patient outcome is most important for this patient? a. The patient will have a diet and exercise plan that results in weight loss. b. The patient will state the reasons for eliminating simple sugars in the diet. c. The patient will have a glycosylated hemoglobin level of less than 7%. d. The patient will choose a diet that distributes calories throughout the day.

C Rationale: The complications of diabetes are related to elevated blood glucose, and the most important patient outcome is the reduction of glucose to near-normal levels. The other outcomes are also appropriate but are not as high in priority. Cognitive Level: Application Text Reference: p. 1273 Nursing Process: Planning NCLEX: Physiological Integrity

A diabetic patient is admitted with ketoacidosis and the health care provider writes all of the following orders. Which order should the nurse implement first? a. Start an infusion of regular insulin at 50 U/hr. b. Give sodium bicarbonate 50 mEq IV push. c. Infuse 1 liter of normal saline per hour. d. Administer regular IV insulin 30 U.

C Rationale: The most urgent patient problem is the hypovolemia associated with DKA, and the priority is to infuse IV fluids. The other actions can be accomplished after the infusion of normal saline is initiated. Cognitive Level: Application Text Reference: p. 1280 Nursing Process: Implementation NCLEX: Physiological Integrity

The patient had a hypoglycemic episode and is treated with a concentrated glucose oral tablet. Fifteen minutes later the capillary glucose level (Accu-Check) is 150 mg/dL. What action should you take? A. Administer a second bolus of glucose solution. B. Administer regular insulin per sliding scale. C. Have the patient eat peanut butter and toast. D. Obtain a serum glucose level.

C. Have the patient eat peanut butter and toast. The patient has had an appropriate response to the glucose. Now a complex carbohydrate is needed to prevent hypoglycemia from reoccurring. There is no need for a second bolus of glucose because the result is within normal range. Insulin is not given, even though the glucose level is slightly elevated. The short-acting glucose is metabolized and insulin administration can increase the risk of a second hypoglycemic reaction. A serum confirmation of the level can be obtained but is not the priority.

The male patient with diabetes and heart disease confides to you that he can no longer have an erection. What is the reason for these changes? A. It is a normal part of aging and is relieved with sildenafil (Viagra). B. It usually is related to emotions and is a temporary problem. C. It is often the first sign of diabetic autonomic neuropathy. D. It indicates that the patient has developed a neurogenic bladder.

C. It is often the first sign of diabetic autonomic neuropathy. Erectile dysfunction (ED) is common and often is the first manifestation of autonomic failure. ED is a common long-term complication of diabetes. Neurogenic bladder is related to urinary retention.

The patient with diabetes reports tingling and burning in the lower extremities at night. The patient asks you why the primary health care provider prescribed the selective serotonin reuptake inhibitor (SSRI) duloxetine (Cymbalta). What is the best response? A. The doctor thought the discomfort was causing the patient to be depressed. B. The drug is known to improve patients' moods and enhance coping. C. It regulates pain by affecting neurotransmitters that transmit pain through the spine. D. It deadens the sensitivity to peripheral nerve endings.

C. It regulates pain by affecting neurotransmitters that transmit pain through the spine. SSRI drugs work by inhibiting the reuptake of norepinephrine and serotonin, which are neurotransmitters that are believed to play a role in the transmission of pain through the spinal cord. Duloxetine is thought to relieve pain by increasing the levels of serotonin and norepinephrine, which improves the body's ability to regulate pain.

** What is a finding in DKA that is not seen in hyperosmolar hyperglycemic syndrome (HHS)? A. Glucose level above 400 mg/dL B. Hyperkalemia C. Ketones in blood D. Urine output of 30 mL/hr

C. Ketones in blood The main difference between the two conditions is that ketone bodies are absent or minimal in HHS because the body has enough insulin to prevent ketoacidosis. Both have high glucose levels, although the level in HHS tends to be higher (above 600 mg/dL). Hypokalemia is possible in both, although it is more likely and serious in DKA. Urine output of 30 mL/hr is normal obligatory output; both conditions are likely to have dehydration and decreased output.

A hospitalized diabetic patient receives 12 U of regular insulin mixed with 34 U of NPH insulin at 7:00 AM. The patient is away from the nursing unit for diagnostic testing at noon, when lunch trays are distributed. The most appropriate action by the nurse is to a. save the lunch tray to be provided upon the patient's return to the unit. b. call the diagnostic testing area and ask that a 5% dextrose IV be started. c. ensure that the patient drinks a glass of milk or orange juice at noon in the diagnostic testing area. d. request that the patient be returned to the unit to eat lunch if testing will not be completed promptly.

D Rationale: Consistency for mealtimes assists with regulation of blood glucose, so the best option is for the patient to have lunch at the usual time. Waiting to eat until after the procedure is likely to cause hypoglycemia. Administration of an IV solution is unnecessarily invasive for the patient. A glass of milk or juice will keep the patient from becoming hypoglycemic but will cause a rapid rise in blood glucose because of the rapid absorption of the simple carbohydrate in these items. Cognitive Level: Analysis Text Reference: p. 1268 Nursing Process: Implementation NCLEX: Physiological Integrity

A patient recovering from DKA asks the nurse how acidosis occurs. The best response by the nurse is that a. insufficient insulin leads to cellular starvation, and as cells rupture they release organic acids into the blood. b. when an insulin deficit causes hyperglycemia, then proteins are deaminated by the liver, causing acidic by-products. c. excess glucose in the blood is metabolized by the liver into acetone, which is acidic. d. an insulin deficit promotes metabolism of fat stores, which produces large amounts of acidic ketones.

D Rationale: Ketoacidosis is caused by the breakdown of fat stores when glucose is not available for intracellular metabolism. The other responses are inaccurate. Cognitive Level: Application Text Reference: pp. 1278-1279 Nursing Process: Implementation NCLEX: Physiological Integrity

A patient with type 1 diabetes has received diet instruction as part of the treatment plan. The nurse determines a need for additional instruction when the patient says, a. "I may have an occasional alcoholic drink if I include it in my meal plan." b. "I will need a bedtime snack because I take an evening dose of NPH insulin." c. "I will eat meals as scheduled, even if I am not hungry, to prevent hypoglycemia." d. "I may eat whatever I want, as long as I use enough insulin to cover the calories."

D Rationale: Most patients with type 1 diabetes need to plan diet choices very carefully. Patients who are using intensified insulin therapy have considerable flexibility in diet choices but still should restrict dietary intake of items such as fat, protein, and alcohol. The other patient statements are correct and indicate good understanding of the diet instruction. Cognitive Level: Application Text Reference: p. 1268 Nursing Process: Evaluation NCLEX: Physiological Integrity

The nurse teaches the diabetic patient who rides a bicycle to work every day to administer morning insulin into the a. thigh. b. buttock. c. arm. d. abdomen.

D Rationale: Patients should be taught not to administer insulin into a site that will be exercised because exercise will increase the rate of absorption. The thigh, buttock, and arm are all exercised by riding a bicycle. Cognitive Level: Application Text Reference: p. 1262 Nursing Process: Implementation NCLEX: Physiological Integrity

A college student who has type 1 diabetes normally walks each evening as part of an exercise regimen. The student now plans to take a swimming class every day at 1:00 PM. The clinic nurse teaches the patient to a. delay eating the noon meal until after the swimming class. b. increase the morning dose of neutral protamine Hagedorn (NPH) insulin on days of the swimming class. c. time the morning insulin injection so that the peak occurs while swimming. d. check glucose level before, during, and after swimming.

D Rationale: The change in exercise will affect blood glucose, and the patient will need to monitor glucose carefully to determine the need for changes in diet and insulin administration. Because exercise tends to decrease blood glucose, patients are advised to eat before exercising. Increasing the morning NPH or timing the insulin to peak during exercise may lead to hypoglycemia, especially with the increased exercise. Cognitive Level: Application Text Reference: p. 1269 Nursing Process: Implementation NCLEX: Health Promotion and Maintenance

A program of weight loss and exercise is recommended for a patient with impaired fasting glucose (IFG). When teaching the patient about the reason for these lifestyle changes, the nurse will tell the patient that a. the high insulin levels associated with this syndrome damage the lining of blood vessels, leading to vascular disease. b. although the fasting plasma glucose levels do not currently indicate diabetes, the glycosylated hemoglobin will be elevated. c. the liver is producing excessive glucose, which will eventually exhaust the ability of the pancreas to produce insulin, and exercise will normalize glucose production. d. the onset of diabetes and the associated cardiovascular risks can be delayed or prevented by weight loss and exercise.

D Rationale: The patient with IFG is at risk for developing type 2 diabetes, but this risk can be decreased with lifestyle changes. Glycosylated hemoglobin levels will not be elevated in IFG and the Hb A1C test is not included in prediabetes testing. Elevated insulin levels do not cause the damage to blood vessels that can occur with IFG. The liver does not produce increased levels of glucose in IFG. Cognitive Level: Application Text Reference: p. 1255 Nursing Process: Implementation NCLEX: Physiological Integrity

A patient screened for diabetes at a clinic has a fasting plasma glucose level of 120 mg/dl (6.7 mmol/L). The nurse will plan to teach the patient about a. use of low doses of regular insulin. b. self-monitoring of blood glucose. c. oral hypoglycemic medications. d. maintenance of a healthy weight.

D Rationale: The patient's impaired fasting glucose indicates prediabetes and the patient should be counseled about lifestyle changes to prevent the development of type 2 diabetes. The patient with prediabetes does not require insulin or the oral hypoglycemics for glucose control and does not need to self-monitor blood glucose. Cognitive Level: Application Text Reference: p. 1255 Nursing Process: Planning NCLEX: Physiological Integrity

The patient in the emergency department is diagnosed with diabetic ketoacidosis. Which laboratory value is essential for you to monitor? A. Magnesium (Mg) B. Hemoglobin (Hb) C. White blood cells (WBCs) D. Potassium (K)

D. Potassium (K) Even if the patient has normal potassium levels, there can be significant hypokalemia when insulin is administered as it pushes the serum potassium intracellularly. This can lead to life-threatening hypokalemia. The other options are not as significant.

Which statement by the patient with type 2 diabetes is accurate. a. "I am supposed to have a meal or snack if I drink alcohol" b. "I am not allowed to eat any sweets because of my diabetes." c. "I do not need to watch what I eat because my diabetes is not the bad kind." d. "The amunt of fat in my diet is not important; it is just the carbohydrates that raise my blood sugar."

a. "I am supposed to have a meal or snack if I drink alcohol" Alcohol should be consumed with food to reduce the risk of hypoglycemia.

The nurse teaches a 38-year-old man who was recently diagnosed with type 1 diabetes mellitus about insulin administration. Which statement by the patient requires an intervention by the nurse? a. "I will discard any insulin bottle that is cloudy in appearance." b. "The best injection site for insulin administration is in my abdomen." c. "I can wash the site with soap and water before insulin administration." d. "I may keep my insulin at room temperature (75o F) for up to a month."

a. "I will discard any insulin bottle that is cloudy in appearance." Intermediate-acting insulin and combination premixed insulin will be cloudy in appearance. Routine hygiene such as washing with soap and rinsing with water is adequate for skin preparation for the patient during self-injections. Insulin vials that the patient is currently using may be left at room temperature for up to 4 weeks unless the room temperature is higher than 86° F (30° C) or below freezing (less than 32° F [0° C]). Rotating sites to different anatomic sites is no longer recommended. Patients should rotate the injection within one particular site, such as the abdomen.

The nurse is assigned to the care of a 64-year-old patient diagnosed with type 2 diabetes. In formulating a teaching plan that encourages the patient to actively participate in management of the diabetes, what should be the nurse's initial intervention? a. Assess patient's perception of what it means to have diabetes. b. Ask the patient to write down current knowledge about diabetes. c. Set goals for the patient to actively participate in managing his diabetes. d. Assume responsibility for all of the patient's care to decrease stress level.

a. Assess patient's perception of what it means to have diabetes. In order for teaching to be effective, the first step is to assess the patient. Teaching can be individualized once the nurse is aware of what a diagnosis of diabetes means to the patient. After the initial assessment, current knowledge can be assessed, and goals should be set with the patient. Assuming responsibility for all of the patient's care will not facilitate the patient's health.

A patient is admitted with diabetes mellitus, malnutrition, and cellulitis. The patient's potassium level is 5.6 mEq/L. The nurse understands that what could be contributing factors for this laboratory result (select all that apply)? a. The level may be increased as a result of dehydration that accompanies hyperglycemia. b. The patient may be excreting extra sodium and retaining potassium because of malnutrition. c. The level is consistent with renal insufficiency that can develop with renal nephropathy. d. The level may be raised as a result of metabolic ketoacidosis caused by hyperglycemia. e. This level demonstrates adequate treatment of the cellulitis and effective serum glucose control.

a. The level may be increased as a result of dehydration that accompanies hyperglycemia. c. The level is consistent with renal insufficiency that can develop with renal nephropathy. d. The level may be raised as a result of metabolic ketoacidosis caused by hyperglycemia. The additional stress of cellulitis may lead to an increase in the patient's serum glucose levels. Dehydration may cause hemoconcentration, resulting in elevated serum readings. Kidneys may have difficulty excreting potassium if renal insufficiency exists. Finally, the nurse must consider the potential for metabolic ketoacidosis since potassium will leave the cell when hydrogen enters in an attempt to compensate for a low pH. Malnutrition does not cause sodium excretion accompanied by potassium retention. Thus it is not a contributing factor to this patient's potassium level. The elevated potassium level does not demonstrate adequate treatment of cellulitis or effective serum glucose control.

The nurse is teaching a 60-year-old woman with type 2 diabetes mellitus how to prevent diabetic nephropathy. Which statement made by the patient indicates that teaching has been successful? a. "Smokeless tobacco products decrease the risk of kidney damage." b. "I can help control my blood pressure by avoiding foods high in salt." c. "I should have yearly dilated eye examinations by an ophthalmologist." d. "I will avoid hypoglycemia by keeping my blood sugar above 180 mg/dL."

b. "I can help control my blood pressure by avoiding foods high in salt." Diabetic nephropathy is a microvascular complication associated with damage to the small blood vessels that supply the glomeruli of the kidney. Risk factors for the development of diabetic nephropathy include hypertension, genetic predisposition, smoking, and chronic hyperglycemia. Patients with diabetes are screened for nephropathy annually with a measurement of the albumin-to-creatinine ratio in urine; a serum creatinine is also needed.

The nurse instructs a 22-year-old female patient with diabetes mellitus about a healthy eating plan. Which statement made by the patient indicates that teaching was successful? a. "I plan to lose 25 pounds this year by following a high-protein diet." b. "I may have a hypoglycemic reaction if I drink alcohol on an empty stomach." c. "I should include more fiber in my diet than a person who does not have diabetes." d. "If I use an insulin pump, I will not need to limit the amount of saturated fat in my diet."

b. "I may have a hypoglycemic reaction if I drink alcohol on an empty stomach." The risk for alcohol-induced hypoglycemia is reduced by eating carbohydrates when drinking alcohol. Intensified insulin therapy, such as the use of an insulin pump, allows considerable flexibility in food selection and can be adjusted for alterations from usual eating and exercise habits. However, saturated fat intake should still be limited to less than 7% of total daily calories. Daily fiber intake of 14 g/1000 kcal is recommended for the general population and for patients with diabetes mellitus. High-protein diets are not recommended for weight loss.

The nurse has taught a patient admitted with diabetes, cellulitis, and osteomyelitis about the principles of foot care. The nurse evaluates that the patient understands the principles of foot care if the patient makes what statement? a. "I should only walk barefoot in nice dry weather." b. "I should look at the condition of my feet every day." c. "I am lucky my shoes fit so nice and tight because they give me firm support." d. "When I am allowed up out of bed, I should check the shower water with my toes."

b. "I should look at the condition of my feet every day." Patients with diabetes mellitus need to inspect their feet daily for broken areas that are at risk for infection and delayed wound healing. Properly fitted (not tight) shoes should be worn at all times. Water temperature should be tested with the hands first.

A 54-year-old patient admitted with type 2 diabetes asks the nurse what "type 2" means. What is the most appropriate response by the nurse? a. "With type 2 diabetes, the body of the pancreas becomes inflamed." b. "With type 2 diabetes, insulin secretion is decreased, and insulin resistance is increased." c. "With type 2 diabetes, the patient is totally dependent on an outside source of insulin." d. "With type 2 diabetes, the body produces autoantibodies that destroy β-cells in the pancreas."

b. "With type 2 diabetes, insulin secretion is decreased, and insulin resistance is increased." In type 2 diabetes mellitus, the secretion of insulin by the pancreas is reduced, and/or the cells of the body become resistant to insulin. The pancreas becomes inflamed with pancreatitis. The patient is totally dependent on exogenous insulin and may have had autoantibodies destroy the β-cells in the pancreas with type 1 diabetes mellitus.

Polydipsia and polyuria related to diabetes mellitus are primarily due to: a. the release of ketones from cells during fat metabolism b. fluid shifts resulting from the osmotic effect of hyperglycemia c. damage to the kidneys from exposure to high levels of glucose. d. changes in RBCs resulting from attachment of excessive glucose to hemoglobin.

b. fluid shifts resulting from the osmotic effect of hyperglycemia

A 65-year-old patient with type 2 diabetes has a urinary tract infection (UTI). The unlicensed assistive personnel (UAP) reported to the nurse that the patient's blood glucose is 642 mg/dL and the patient is hard to arouse. When the nurse assesses the urine, there are no ketones present. What collaborative care should the nurse expect for this patient? a. Routine insulin therapy and exercise b. Administer a different antibiotic for the UTI. c. Cardiac monitoring to detect potassium changes d. Administer IV fluids rapidly to correct dehydration.

c. Cardiac monitoring to detect potassium changes This patient has manifestations of hyperosmolar hyperglycemic syndrome (HHS). Cardiac monitoring will be needed because of the changes in the potassium level related to fluid and insulin therapy and the osmotic diuresis from the elevated serum glucose level. Routine insulin would not be enough, and exercise could be dangerous for this patient. Extra insulin will be needed. The type of antibiotic will not affect HHS. There will be a large amount of IV fluid administered, but it will be given slowly because this patient is older and may have cardiac or renal compromise requiring hemodynamic monitoring to avoid fluid overload during fluid replacement.

A college student is newly diagnosed with type 1 diabetes. She now has a headache, changes in her vision, and is anxious, but does not have her portable blood glucose monitor with her. Which action should the campus nurse advise her to take? a. Eat a piece of pizza. b. Drink some diet pop. c. Eat 15 g of simple carbohydrates. d. Take an extra dose of rapid-acting insulin.

c. Eat 15 g of simple carbohydrates. When the patient with type 1 diabetes is unsure about the meaning of the symptoms she is experiencing, she should treat herself for hypoglycemia to prevent seizures and coma from occurring. She should also be advised to check her blood glucose as soon as possible. The fat in the pizza and the diet pop would not allow the blood glucose to increase to eliminate the symptoms. The extra dose of rapid-acting insulin would further decrease her blood glucose.

A patient, who is admitted with diabetes mellitus, has a glucose level of 380 mg/dL and a moderate level of ketones in the urine. As the nurse assesses for signs of ketoacidosis, which respiratory pattern would the nurse expect to find? a. Central apnea b. Hypoventilation c. Kussmaul respirations d. Cheyne-Stokes respirations

c. Kussmaul respirations In diabetic ketoacidosis, the lungs try to compensate for the acidosis by blowing off volatile acids and carbon dioxide. This leads to a pattern of Kussmaul respirations, which are deep and nonlabored. Central apnea occurs because the brain temporarily stops sending signals to the muscles that control breathing, which is unrelated to ketoacidosis. Hypoventilation and Cheyne-Stokes respirations do not occur with ketoacidosis.

The newly diagnosed patient with type 2 diabetes has been prescribed metformin (Glucophage). What should the nurse tell the patient to best explain how this medication works? a. Increases insulin production from the pancreas. b. Slows the absorption of carbohydrate in the small intestine. c. Reduces glucose production by the liver and enhances insulin sensitivity. d. Increases insulin release from the pancreas, inhibits glucagon secretion, and decreases gastric emptying.

c. Reduces glucose production by the liver and enhances insulin sensitivity. Metformin is a biguanide that reduces glucose production by the liver and enhances the tissue's insulin sensitivity. Sulfonylureas and meglitinides increase insulin production from the pancreas. α-glucosidase inhibitors slow the absorption of carbohydrate in the intestine. Glucagon-like peptide receptor agonists increase insulin synthesis and release from the pancreas, inhibit glucagon secretion, and decrease gastric emptying.

The nurse is teaching a patient with type 2 diabetes mellitus about exercise to help control his blood glucose. The nurse knows the patient understands when the patient elicits which exercise plan? a. "I want to go fishing for 30 minutes each day; I will drink fluids and wear sunscreen." b. "I will go running each day when my blood sugar is too high to bring it back to normal." c. "I will plan to keep my job as a teacher because I get a lot of exercise every school day." d. "I will take a brisk 30-minute walk 5 days per week and do resistance training 3 times a week."

d. "I will take a brisk 30-minute walk 5 days per week and do resistance training 3 times a week." The best exercise plan for the person with type 2 diabetes is for 30 minutes of moderate activity 5 days per week and resistance training 3 times a week. Brisk walking is moderate activity. Fishing and teaching are light activity, and running is considered vigorous activity.

The nurse caring for a patient hospitalized with diabetes mellitus would look for which laboratory test result to obtain information on the patient's past glucose control? a. Prealbumin level b. Urine ketone level c. Fasting glucose level d. Glycosylated hemoglobin level

d. Glycosylated hemoglobin level A glycosylated hemoglobin level detects the amount of glucose that is bound to red blood cells (RBCs). When circulating glucose levels are high, glucose attaches to the RBCs and remains there for the life of the blood cell, which is approximately 120 days. Thus the test can give an indication of glycemic control over approximately 2 to 3 months. The prealbumin level is used to establish nutritional status and is unrelated to past glucose control. The urine ketone level will only show that hyperglycemia or starvation is probably currently occurring. The fasting glucose level only indicates current glucose control.


Conjuntos de estudio relacionados

Psych 332 - Adolescence Psych Quiz Chapter #12

View Set

Quiz 3 Violence and Injury/ Psychological health

View Set

Intermediate Accounting III ch 20

View Set

Renaissance to Modern Art Midterm

View Set

Chapter 5 ITN 200 Configuring Resource Access

View Set

Configuring a basic Apache Server

View Set